9
$\begingroup$

The question is as in the title:

Is there a nonpolynomial $C^\infty$ function $f$ on $\mathbb{R}$ such that $\sup_{x \in \mathbb{R}} \lvert f^{(q)}(x) \rvert \leq (\ln q)^{-q}$ for every natural number $q >1$?

Here, "nonpolynomial" excludes constant functions as well, of course.

I think nonpolynomial functions "very uniformly approximating" a constant function" might satisfy such a bound, but I cannot find such a nice example.

Could anyone please help me?

$\endgroup$

2 Answers 2

17
$\begingroup$

The answer is no. From Taylor's theorem with remainder, we see that for any integer $q>2$, we have $$ f^{(2)}(x) = \sum_{j=0}^{q-3} \frac{f^{(2+j)}(0)}{j!} x^j + O( \frac{(\ln q)^{-q} |x|^{q-2}}{(q-2)!} )$$ and thus for $|x| \leq \frac{q \ln q}{10}$ (say) we have from Stirling's formula that $$ \sum_{j=0}^{q-3} \frac{f^{(2+j)}(0)}{j!} x^j = O( 1 ).$$ The key point here is that we have bounded control on a polynomial on an interval of length much wider than its degree (a situation in which one expects favorable estimates thanks to the uncertainty principle). Making the trigonometric substitution $x = \frac{q \ln q}{10} \sin \theta$ we conclude $$ \sum_{j=0}^{q-3} \frac{f^{(2+j)}(0)}{j!} (\frac{q \ln q}{10})^j \sin^j \theta = O( 1 )$$ for all $\theta$. The LHS is a trigonometric polynomial of degree $O(q)$, so by the Bernstein inequality we have $$ \frac{d}{d\theta} \sum_{j=0}^{q-3} \frac{f^{(2+j)}(0)}{j!} (\frac{q \ln q}{10})^j \sin^j \theta = O( q )$$ for all $\theta$. Evaluating this at $\theta=0$ we conclude that $$ f^{(3)}(0) \frac{q \ln q}{10} = O(q).$$ Dividing by $q \ln q$ and sending $q \to \infty$ we conclude that $f^{(3)}(0)$ vanishes. Applying a translation to this argument, we see that $f^{(3)}$ vanishes everywhere, thus $f$ is a polynomial (in fact it is quadratic).

$\endgroup$
5
  • 1
    $\begingroup$ Thank you for your wonderful answer. What I am looking for is a smooth function $f(x)$ whose $q^{th}$ order derivative $f^{(q)}$ is uniformly bounded by some $A(q)^{-q}$ where $A(q) \to \infty$ as $q \to \infty$. I just chose $A(q)=\log q$ but it does not seem to work. Is there such $A(q)$ and nonpolynomial $f(x)$? Could you please help me once more? $\endgroup$
    – Isaac
    Jan 18 at 0:10
  • 4
    $\begingroup$ @Isaac Terry's argument does not change and remains equally valid if you mechanically replace every instance of $\ln q$ by $A(q)$ in it. $\endgroup$
    – fedja
    Jan 18 at 0:33
  • $\begingroup$ I see. Thank you for point that out. I reformulated my question to be a bit more "realistic". Could you please help me? : mathoverflow.net/questions/438766/… $\endgroup$
    – Isaac
    Jan 18 at 2:16
  • $\begingroup$ "we have bounded control on a polynomial on an interval of length much wider than its degree (a situation in which one expects favorable estimates thanks to the uncertainty principle)" - Do you have a reference for this version of the uncertainty principle? Or maybe how this fits into some general uncertainty principle heuristics? $\endgroup$ Jan 19 at 1:41
  • 1
    $\begingroup$ The usual Fourier uncertainty principle tells us that trig polynomials of degree $n$ behave roughly like constants at spatial scales of length $1/n$. After a trig substitution, this implies that polynomials of degree n that are controlled on an interval of length $I$ should be have roughly like constants at scales $|I|/n$, at least in the bulk of the interval (and with errors measured relative to the size of the polynomial on $I$). So if $|I|/n$ can be made to go to infinity, one can hope to force the polynomial to be constant, as is done in my argument here. $\endgroup$
    – Terry Tao
    Jan 19 at 18:26
4
$\begingroup$

$\newcommand{\R}{\mathbb R}\newcommand\ep\varepsilon\newcommand{\de}{\delta}$Let us prove the following weaker statement, hoping it would help to find a complete solution of the posted problem.

Proposition 1: Suppose that a function $f\in C^\infty(\R)$ is such that \begin{equation*} |f^{(k)}|\le2^{-c_k 2^k} \tag{1}\label{1} \end{equation*} for all $k=0,1,\dots$, where $c_k\to\infty$ (as $k\to\infty$). Then $f$ is constant.

The proof of Proposition 1 is based on

Lemma 1: Suppose that $g\in C^2(\R)$, $|g|\le B\in(0,\infty)$, and $|g'(a)|\ge C\in(0,\infty)$ for some real $a$. Then \begin{equation*} |g''(x)|\ge D:=\frac{C^2}{2B} \end{equation*} for some real $x$.

Proof of Lemma 1: To obtain a contradiction, suppose that $|g''|<D$. Using the vertical and/or horizontal reflections of the graph of $g$, without loss of generality (wlog) assume that $g(a)\ge0$ and $g'(a)\ge0$, so that $g'(a)\ge C>0$. So, for any real $b>0$, \begin{equation*} g(a+b)>g(a)+g'(a)b-Db^2/2\ge0+Cb-Db^2/2=B \end{equation*} if $b=2B/C$, which contradicts condition $|g|\le B$. $\quad\Box$

Proof of Proposition 1: To obtain a contradiction, suppose that $\ep_1:=|f'(a_1)|>0$ for some real $a_1$. By \eqref{1}, $|f^{(k)}|\le B$ for some real $B\ge\ep_1$ and all $k=0,1,\dots$. So, by Lemma 1, \begin{equation*} |f''(a_2)|\ge\ep_2:=\frac{\ep_1^2}{2B} \end{equation*} for some real $a_2$. Continuing so, we get a sequence $(\ep_1,\ep_2,\dots)$ of positive real numbers and a sequence $(a_1,a_2,\dots)$ of real numbers such that for all $k=1,2,\dots$ \begin{equation*} |f^{(k)}(a_k)|\ge\ep_k:=\frac{\ep_{k-1}^2}{2B} \end{equation*} and hence, with $c:=\frac12\log_2\frac{2B}{\ep_1}\in(0,\infty)$, \begin{equation*} |f^{(k)}(a_k)|\ge\ep_k=2^{-c2^k}, \end{equation*} which contradicts \eqref{1}. $\quad\Box$

$\endgroup$

Your Answer

By clicking “Post Your Answer”, you agree to our terms of service and acknowledge that you have read and understand our privacy policy and code of conduct.

Not the answer you're looking for? Browse other questions tagged or ask your own question.